1. A sequence x(n) with the z transform X(z) = z4 + z2 - 2z + 2 - 3z - 4 is applied as an input to a linear, time invariant system with the impulse response h(n) = 2δ(n - 3) where The output at n = 4 is





Ask Your Doubts Here

Type in
(Press Ctrl+g to toggle between English and the chosen language)

Comments

  • By: guest on 01 Jun 2017 11.49 pm
    Y(Z) = X(Z) x H(Z) = 2 x (Z4 + Z2 - 2Z + 2) x Z-3 Y(n) = 2[δ(n + 1) - δ(n - 1) - 2(n - 2) + 2δ(n - 3) - 3δ(n - 7)] at n = 4, Y(4) = 0.
Show Similar Question And Answers
QA->Impulse of force?....
QA->Coefficient of linear expansion always …….. with the increase in temperature?....
QA->Which ministry signed a Memorandum of Understanding (MoU) with National Council of Applied Economic Research for improving the official statistical system in the country?....
QA->After the attainment of political independence in 1947, Gandhiji felt that the Congress, as a propaganda vehicle and a parliamentary machine, had outlived its usefulness. So to keep the Congress away from unhealthy competition with political parties and communal bodies, Gandhiji towards the end of January 1948 sketched a draft constitution for the Congress to transform itself into ......?....
QA->ഇന്ത്യയിലെ ആദ്യ ബാഡ്‌മിന്റൺ ബ്രാന്റായ Transform ന്റെ ബ്രാൻഡ് അംബാസിഡറായി നിയമിതനായത്?....
MCQ->A sequence x(n) with the z transform X(z) = z4 + z2 - 2z + 2 - 3z - 4 is applied as an input to a linear, time invariant system with the impulse response h(n) = 2δ(n - 3) where The output at n = 4 is....
MCQ->A linear time-invariant, causal continuous time system has a rational transfer function with simple poles at s = - 2 and s = - 4, and one simple zero at s = - 1. A unit step u(t) is applied at the input of the system. At steady state, the output has constant value of 1. The impulse response of this system is....
MCQ->A discrete time linear shift-invariant system has an impulse response h[n] with h[0] = 1, h[1] = - 1, h[2] - 2, and zero otherwise. The system is given an input sequence x[n] with x[0] - x[2] - 1, and zero otherwise. The number of nonzero samples in the output sequence y[n], and the value of y[2] are, respectively....
MCQ->The response of a linea, time invariant discrete time system to a unit step input ∪(n) is the unit impulse δ(n). The system response to a ramp input n ∪(n) would be....
MCQ->If h1[n] = 3δ[n] + δ[n - 1], h2[n] = 2δ[n] + δ[n - 2] h3[n] = δ[n] - 3δ[n - 1] + 7δ[n - 4] + 6δ[n - 6] h[n] = h1[n] h2[n] + h3[n] h1[n] has value....
Terms And Service:We do not guarantee the accuracy of available data ..We Provide Information On Public Data.. Please consult an expert before using this data for commercial or personal use | Powered By:Omega Web Solutions
© 2002-2017 Omega Education PVT LTD...Privacy | Terms And Conditions
Question ANSWER With Solution